Difference between revisions of "User:Nik"

From Ferienserie MMP2
Jump to: navigation, search
(Comment about the chain rule at the end of solution II)
(Solution III: I changed it such that we don't have to use the wrong chain rule at the end)
Line 86: Line 86:
  
 
$$ \{P_i, P_j\} = \sum_{k=1}^{n}\left(\frac{\partial P_i}{\partial q_k}\frac{\partial P_j}{\partial p_k} - \frac{\partial P_i}{\partial p_k}\frac{\partial P_j}{\partial q_k}\right) $$
 
$$ \{P_i, P_j\} = \sum_{k=1}^{n}\left(\frac{\partial P_i}{\partial q_k}\frac{\partial P_j}{\partial p_k} - \frac{\partial P_i}{\partial p_k}\frac{\partial P_j}{\partial q_k}\right) $$
Now we put in the results from equation (3) and (4)
+
Now we put in the results from equation (3) and (4) in all 4 parts of the sum
$$ = \sum_{k=1}^{n}\left(\frac{\partial P_i}{\partial q_k}\left(-\sum_{s=1}^{n}\left(\frac{\partial^2 \Phi}{\partial Q_j \partial Q_s}(q, Q)\frac{\partial Q_s}{\partial p_k}\right)\right) - \frac{\partial P_i}{\partial p_k}\left(-\frac{\partial^2 \Phi}{\partial Q_j \partial q_k}(q, Q) - \sum_{s=1}^{n}\left(\frac{\partial^2 \Phi}{\partial Q_j \partial Q_s}(q, Q)\frac{\partial Q_s}{\partial q_k}\right)\right)\right) $$
+
$$ = \sum_{k=1}^{n}\left(\left(-\frac{\partial^2 \Phi}{\partial Q_i \partial q_k}(q, Q) - \underbrace{\sum_{s=1}^{n}\left(\frac{\partial^2 \Phi}{\partial Q_i \partial Q_s}(q, Q)\frac{\partial Q_s}{\partial q_k}\right)}_{(a)}\right)\left(\underbrace{-\sum_{s=1}^{n}\left(\frac{\partial^2 \Phi}{\partial Q_j \partial Q_s}(q, Q)\frac{\partial Q_s}{\partial p_k}\right)}_{(b)}\right) \\ - \left(\underbrace{-\sum_{s=1}^{n}\left(\frac{\partial^2 \Phi}{\partial Q_i \partial Q_s}(q, Q)\frac{\partial Q_s}{\partial p_k}\right)}_{(c)}\right)\left(-\frac{\partial^2 \Phi}{\partial Q_j \partial q_k}(q, Q) - \underbrace{\sum_{s=1}^{n}\left(\frac{\partial^2 \Phi}{\partial Q_j \partial Q_s}(q, Q)\frac{\partial Q_s}{\partial q_k}\right)}_{(d)}\right)\right) $$
Now we multiply out and separate all the sums. We can rearrange it like this because all the parts we put inside are independent on the running variable.
+
If we multiply out we see that (a) * (b) = (c) * (d) and because of the sign they cancel out.
$$ =  \sum_{k=1}^{n}\left(\frac{\partial P_i}{\partial p_k}\frac{\partial^2 \Phi}{\partial Q_j \partial q_k}(q, Q)\right) + \sum_{k=1}^{n}\sum_{s=1}^{n}\left(\frac{\partial P_i}{\partial p_k}\frac{\partial^2 \Phi}{\partial Q_j \partial Q_s}(q, Q)\frac{\partial Q_s}{\partial q_k}\right) - \sum_{k=1}^{n}\sum_{s=1}^{n}\left(\frac{\partial P_i}{\partial q_k}\frac{\partial^2 \Phi}{\partial Q_j \partial Q_s}(q, Q)\frac{\partial Q_s}{\partial p_k}\right) $$
+
So we multiply out the remaining parts
Now we rearrange again such that we can build a new Poisson bracket
+
$$ = \sum_{k=1}^{n}\sum_{s=1}^{n}\left(\frac{\partial^2 \Phi}{\partial Q_i \partial q_k}(q, Q)\frac{\partial^2 \Phi}{\partial Q_j \partial Q_s}(q, Q)\frac{\partial Q_s}{\partial p_k} - \frac{\partial^2 \Phi}{\partial Q_i \partial Q_s}(q, Q)\frac{\partial Q_s}{\partial p_k}\frac{\partial^2 \Phi}{\partial Q_j \partial q_k}(q, Q)\right) $$
$$ =  \sum_{k=1}^{n}\left(\frac{\partial P_i}{\partial p_k}\frac{\partial^2 \Phi}{\partial Q_j \partial q_k}(q, Q)\right) + \sum_{s=1}^{n}\left(\frac{\partial^2 \Phi}{\partial Q_j \partial Q_s}(q, Q) \underbrace{\sum_{k=1}^{n}\left(\frac{\partial P_i}{\partial p_k}\frac{\partial Q_s}{\partial q_k} - \frac{\partial P_i}{\partial q_k}\frac{\partial Q_s}{\partial p_k}\right)}_{=\{Q_s, P_i\}=\delta_{si}}\right) $$
+
And use the definition from the problem set
Now we use the definition from the problem and take the \( \delta \) out.
+
$$ = \sum_{k=1}^{n}\sum_{s=1}^{n}\left(-\frac{\partial^2 \Phi}{\partial Q_i \partial q_k}(q, Q)\underbrace{\frac{\partial P_s}{\partial Q_j}}_{=0}\frac{\partial Q_s}{\partial p_k} + \underbrace{\frac{\partial P_s}{\partial Q_i}}_{=0}\frac{\partial Q_s}{\partial p_k}\frac{\partial^2 \Phi}{\partial Q_j \partial q_k}(q, Q)\right) $$
$$ =  \sum_{k=1}^{n}\left(\frac{\partial P_i}{\partial p_k}\frac{\partial p_k}{\partial Q_j}\right) + \frac{\partial^2 \Phi}{\partial Q_j \partial Q_i}(q, Q) $$
+
 
Now we use the chain rule and the definition again
+
 
$$ = \frac{\partial P_i}{\partial Q_j} - \frac{\partial P_i}{\partial Q_j} = 0 $$
+
 
 +
$$ = 0 $$

Revision as of 13:45, 16 January 2015

Foreword

I use \(Q\:/\:P\) for the transformed system instead of \(\widetilde{q}\:/\:\widetilde{p}\) because it's easier to write in Latex.

Problem

Let \( \Phi \in C^\infty(\mathbb{R}^{2n}) \) have the property that the system \( p_i = \frac{\partial}{\partial q_i} \Phi (q, Q) \) has a unique smooth solution \( Q = Q(q,p) \).

Define \( P_i(q,p) = - \frac{\partial}{\partial Q_i} \Phi (q, Q) | _{Q= Q(q,p)} \)

Let \( \{\cdot,\cdot\} \) be the Poisson bracket, such that \( \{f,g\} = \sum_{j=1}^n \frac{\partial f}{\partial q_j} \frac{\partial g}{\partial p_j} - \frac{\partial f}{\partial p_j} \frac{\partial g}{\partial q_j} \)

Show that:

I) \( \{Q_i(q,p), Q_j(q,p)\} = 0 \)

II) \( \{Q_i(q,p), P_j(q,p)\} = \delta_{ij} \)

III) \( \{P_i (q,p), P_j(q,p)\} = 0 \)


Solution

Important equations

$$ \underbrace{\frac{\partial p_i}{\partial q_k}}_{0} = \frac{\partial}{\partial q_k}\left(\frac{\partial \Phi}{\partial q_i}(q, Q)\right) $$ $$ = \sum_{s=1}^{n}\left(\frac{\partial^2 \Phi}{\partial q_i \partial q_s}(q, Q)\underbrace{\frac{\partial q_s}{\partial q_k}}_{\delta_{sk}} + \frac{\partial^2 \Phi}{\partial q_i \partial Q_s}(q, Q)\frac{\partial Q_s}{\partial q_k}\right) $$ $$ \Rightarrow 0 = \frac{\partial^2 \Phi}{\partial q_i \partial q_k}(q, Q) + \sum_{s=1}^{n}\left(\frac{\partial^2 \Phi}{\partial q_i \partial Q_s}(q, Q)\frac{\partial Q_s}{\partial q_k}\right) $$ $$ \tag{1} \Rightarrow \boxed{ -\frac{\partial^2 \Phi}{\partial q_i \partial q_k}(q, Q) = \sum_{s=1}^{n}\left(\frac{\partial^2 \Phi}{\partial q_i \partial Q_s}(q, Q)\frac{\partial Q_s}{\partial q_k}\right) } $$


$$ \underbrace{\frac{\partial p_i}{\partial p_k}}_{\delta_{ik}} = \frac{\partial}{\partial p_k}\left(\frac{\partial \Phi}{\partial q_i}(q, Q)\right) $$ $$ = \sum_{s=1}^{n}\left(\frac{\partial^2 \Phi}{\partial q_i \partial q_s}(q, Q)\underbrace{\frac{\partial q_s}{\partial p_k}}_{0} + \frac{\partial^2 \Phi}{\partial q_i \partial Q_s}(q, Q)\frac{\partial Q_s}{\partial p_k}\right) $$ $$ \tag{2} \boxed{ \Rightarrow \delta_{ik} = \sum_{s=1}^{n}\left(\frac{\partial^2 \Phi}{\partial q_i \partial Q_s}(q, Q)\frac{\partial Q_s}{\partial p_k}\right) } $$


$$ \frac{\partial P_i}{\partial q_k} = \frac{\partial}{\partial q_k}\left(-\frac{\partial \Phi}{\partial Q_i}(q, Q)\right) $$ $$ = \sum_{s=1}^{n}\left(-\frac{\partial^2 \Phi}{\partial Q_i \partial q_s}(q, Q)\underbrace{\frac{\partial q_s}{\partial q_k}}_{\delta_{sk}} - \frac{\partial^2 \Phi}{\partial Q_i \partial Q_s}(q, Q)\frac{\partial Q_s}{\partial q_k}\right) $$ $$ \tag{3} \boxed{ \Rightarrow \frac{\partial P_i}{\partial q_k} = -\frac{\partial^2 \Phi}{\partial Q_i \partial q_k}(q, Q) - \sum_{s=1}^{n}\left(\frac{\partial^2 \Phi}{\partial Q_i \partial Q_s}(q, Q)\frac{\partial Q_s}{\partial q_k}\right) } $$


$$ \frac{\partial P_i}{\partial p_k} = \frac{\partial}{\partial p_k}\left(-\frac{\partial \Phi}{\partial Q_i}(q, Q)\right) $$ $$ = \sum_{s=1}^{n}\left(-\frac{\partial^2 \Phi}{\partial Q_i \partial q_s}(q, Q)\underbrace{\frac{\partial q_s}{\partial p_k}}_{0} - \frac{\partial^2 \Phi}{\partial Q_i \partial Q_s}(q, Q)\frac{\partial Q_s}{\partial p_k}\right) $$ $$ \tag{4} \Rightarrow \boxed{ \frac{\partial P_i}{\partial p_k} = -\sum_{s=1}^{n}\left(\frac{\partial^2 \Phi}{\partial Q_i \partial Q_s}(q, Q)\frac{\partial Q_s}{\partial p_k}\right) } $$


Solution I)

This is the most complicated problem but we calculate it first because we use it in II) und III) First we write equations (1) and (2) again an shift our eyeballs between them and look at it as matrix multiplication: $$ \underbrace{-\frac{\partial^2 \Phi}{\partial q_i \partial q_k}(q, Q)}_{D_{ik}} = \sum_{s=1}^{n}\left(\underbrace{\frac{\partial^2 \Phi}{\partial q_i \partial Q_s}(q, Q)}_{A_{is}}\underbrace{\frac{\partial Q_s}{\partial q_k}}_{C_{sk}}\right) $$ $$ \underbrace{\delta_{ik}}_{\mathbb{1}} = \sum_{s=1}^{n}\left(\underbrace{\frac{\partial^2 \Phi}{\partial q_i \partial Q_s}(q, Q)}_{A_{is}}\underbrace{\frac{\partial Q_s}{\partial p_k}}_{B_{sk}}\right) $$ Now let's do some linear algebra: We know that \( \mathbb{1} = AB \Rightarrow A^{-1} = B \text{ and } D = AC \) $$ \Rightarrow A^{-1}D = C $$ $$\Rightarrow BD = C $$ With that we can now calculate C which we need to calculate the Poisson bracket $$ C_{sk} = - \sum_{e=1}^{n}\left(\frac{\partial Q_s}{\partial p_e}\frac{\partial^2 \Phi}{\partial q_k \partial q_e}\right) $$ Now we change the indices such that it fits into the equation $$ \frac{\partial Q_i}{\partial q_k} = - \sum_{s=1}^{n}\left(\frac{\partial Q_i}{\partial p_s}\frac{\partial^2 \Phi}{\partial q_k \partial q_s}\right) $$ Finally we are prepared to calculate the Poisson bracket $$ \{Q_i, Q_j\} = \sum_{k=1}^{n}\left(\frac{\partial Q_i}{\partial q_k}\frac{\partial Q_j}{\partial p_k} - \frac{\partial Q_i}{\partial p_k}\frac{\partial Q_j}{\partial q_k}\right) $$ Put in the sum from above $$ = \sum_{k=1}^{n}\left(\frac{\partial Q_j}{\partial p_k}\left(-\sum_{s=1}^{n}\left(\frac{\partial Q_i}{\partial p_s}\frac{\partial^2 \Phi}{\partial q_k \partial q_s}(q, Q)\right)\right) - \frac{\partial Q_i}{\partial p_k}\left(-\sum_{s=1}^{n}\left(\frac{\partial Q_j}{\partial p_s}\frac{\partial^2 \Phi}{\partial q_k \partial q_s}(q, Q)\right)\right)\right) $$ Take the sum out $$ \sum_{k=1}^{n}\sum_{s=1}^{n}\left(\frac{\partial Q_i}{\partial p_k}\frac{\partial Q_j}{\partial p_s}\frac{\partial^2 \Phi}{\partial q_k \partial q_s}(q, Q) - \frac{\partial Q_j}{\partial p_k}\frac{\partial Q_i}{\partial p_s}\frac{\partial^2 \Phi}{\partial q_k \partial q_s}(q, Q) \right) = 0 $$ Since we sum with \(s\) and \(k\) completely from \(1\) to \(n\) the two parts are equal and therefore the difference is zero.

Solution II)

$$ \{Q_i, P_j\} = \sum_{k=1}^{n}\left( \frac{\partial Q_i}{\partial q_k}\frac{\partial P_j}{\partial p_k} - \frac{\partial Q_i}{\partial p_k}\frac{\partial P_j}{\partial q_k} \right) $$ Now we put in the results from equation (3) and (4) $$ = \sum_{k=1}^{n}\left(\frac{\partial Q_i}{\partial q_k}\left(-\sum_{s=1}^{n}\left(\frac{\partial^2 \Phi}{\partial Q_j \partial Q_s}(q, Q)\frac{\partial Q_s}{\partial p_k}\right)\right) - \frac{\partial Q_i}{\partial p_k}\left(-\frac{\partial^2 \Phi}{\partial Q_j \partial q_k}(q, Q) - \sum_{s=1}^{n}\left(\frac{\partial^2 \Phi}{\partial Q_j \partial Q_s}(q, Q)\frac{\partial Q_s}{\partial q_k}\right)\right)\right) $$ Now we multiply out and separate all the sums. We can rearrange it like this because all the parts we put inside are independent on the running variable. $$ = \sum_{k=1}^{n}\left(\frac{\partial Q_i}{\partial p_k}\frac{\partial^2 \Phi}{\partial Q_j \partial q_k}(q, Q)\right) + \sum_{k=1}^{n}\sum_{s=1}^{n}\left(\frac{\partial Q_i}{\partial p_k}\frac{\partial^2 \Phi}{\partial Q_j \partial Q_s}(q, Q)\frac{\partial Q_s}{\partial q_k}\right) - \sum_{k=1}^{n}\sum_{s=1}^{n}\left(\frac{\partial Q_i}{\partial q_k}\frac{\partial^2 \Phi}{\partial Q_j \partial Q_s}(q, Q)\frac{\partial Q_s}{\partial p_k}\right) $$ Now we rearrange again such that we can build a new Poisson bracket $$ = \underbrace{\sum_{k=1}^{n}\left(\frac{\partial Q_i}{\partial p_k}\frac{\partial^2 \Phi}{\partial Q_j \partial q_k}(q, Q)\right)}_{\color{red}{\delta_{ij}}} + \sum_{s=1}^{n}\left(\frac{\partial^2 \Phi}{\partial Q_j \partial Q_s}(q, Q) \underbrace{\sum_{k=1}^{n}\left(\frac{\partial Q_i}{\partial p_k}\frac{\partial Q_s}{\partial q_k} - \frac{\partial Q_i}{\partial q_k}\frac{\partial Q_s}{\partial p_k}\right)}_{=\{Q_s, Q_i\}=0}\right) $$ Now we use the definition from the problem and then the chain rule $$ = \sum_{k=1}^{n}\left(\frac{\partial Q_i}{\partial p_k}\frac{\partial p_k}{\partial Q_j} \right) = \frac{\partial Q_i}{\partial Q_j} = \delta_{ij} $$ The chain rule is here not correct. It would imply that \( Q \) is independent of \( q \) and that is not correct. Since I got no better solution I would write \( \delta_{ij} \) directly in the sum above.

Solution III)

$$ \{P_i, P_j\} = \sum_{k=1}^{n}\left(\frac{\partial P_i}{\partial q_k}\frac{\partial P_j}{\partial p_k} - \frac{\partial P_i}{\partial p_k}\frac{\partial P_j}{\partial q_k}\right) $$ Now we put in the results from equation (3) and (4) in all 4 parts of the sum $$ = \sum_{k=1}^{n}\left(\left(-\frac{\partial^2 \Phi}{\partial Q_i \partial q_k}(q, Q) - \underbrace{\sum_{s=1}^{n}\left(\frac{\partial^2 \Phi}{\partial Q_i \partial Q_s}(q, Q)\frac{\partial Q_s}{\partial q_k}\right)}_{(a)}\right)\left(\underbrace{-\sum_{s=1}^{n}\left(\frac{\partial^2 \Phi}{\partial Q_j \partial Q_s}(q, Q)\frac{\partial Q_s}{\partial p_k}\right)}_{(b)}\right) \\ - \left(\underbrace{-\sum_{s=1}^{n}\left(\frac{\partial^2 \Phi}{\partial Q_i \partial Q_s}(q, Q)\frac{\partial Q_s}{\partial p_k}\right)}_{(c)}\right)\left(-\frac{\partial^2 \Phi}{\partial Q_j \partial q_k}(q, Q) - \underbrace{\sum_{s=1}^{n}\left(\frac{\partial^2 \Phi}{\partial Q_j \partial Q_s}(q, Q)\frac{\partial Q_s}{\partial q_k}\right)}_{(d)}\right)\right) $$ If we multiply out we see that (a) * (b) = (c) * (d) and because of the sign they cancel out. So we multiply out the remaining parts $$ = \sum_{k=1}^{n}\sum_{s=1}^{n}\left(\frac{\partial^2 \Phi}{\partial Q_i \partial q_k}(q, Q)\frac{\partial^2 \Phi}{\partial Q_j \partial Q_s}(q, Q)\frac{\partial Q_s}{\partial p_k} - \frac{\partial^2 \Phi}{\partial Q_i \partial Q_s}(q, Q)\frac{\partial Q_s}{\partial p_k}\frac{\partial^2 \Phi}{\partial Q_j \partial q_k}(q, Q)\right) $$ And use the definition from the problem set $$ = \sum_{k=1}^{n}\sum_{s=1}^{n}\left(-\frac{\partial^2 \Phi}{\partial Q_i \partial q_k}(q, Q)\underbrace{\frac{\partial P_s}{\partial Q_j}}_{=0}\frac{\partial Q_s}{\partial p_k} + \underbrace{\frac{\partial P_s}{\partial Q_i}}_{=0}\frac{\partial Q_s}{\partial p_k}\frac{\partial^2 \Phi}{\partial Q_j \partial q_k}(q, Q)\right) $$


$$ = 0 $$